LSAT and Law School Admissions Forum

Get expert LSAT preparation and law school admissions advice from PowerScore Test Preparation.

 Administrator
PowerScore Staff
  • PowerScore Staff
  • Posts: 8915
  • Joined: Feb 02, 2011
|
#23471
Complete Question Explanation

Weaken—CE, #%. The correct answer choice is (E)

In this stimulus the police commissioner cites a 15% decrease in violent crime, and the fact that mandatory sentencing enactment was the only major policy change made last year. Based on these facts, the commissioner concludes that the drop in crime rate must be attributable to the sentencing law. Since this causal stimulus is followed by a weaken question, we might start by looking for an alternative cause for the 15% decrease in the violent crime rate.

Answer choice (A): Since we have no information regarding the economic conditions, this choice has no effect on the strength of the author's argument.

Answer choice (B): This wouldn't change the fact that the referenced enactment was the only major policy change and that it was followed by a decrease in the violent crime rate.

Answer choice (C): The overall crime rate has no effect on this argument, which concerns itself only with the violent crime rate.

Answer choice (D): If the definition became broader (thereby making it easier to be found guilty of a violent offense, then the decrease in the violent crime rate is even more surprising. This would not weaken the argument, however, regarding the cause of the decrease, so this answer choice is incorrect.

Answer choice (E): This is the correct answer choice. If this is the case, then older policy changes had some lag time before their effects could be felt. Since this answer choice provides another possible cause for the violent crime rate decrease, this is the correct answer.
 allik@umich.edu
  • Posts: 5
  • Joined: May 03, 2016
|
#24045
I am having trouble understanding why (E) provides an alternate cause. Why does more police decrease the actual crime rate? If anything it would just increase the rate of arrest.
 Robert Carroll
PowerScore Staff
  • PowerScore Staff
  • Posts: 1787
  • Joined: Dec 06, 2013
|
#24090
allik,

Because the police commissioner is claiming that the mandatory sentencing law is the only possible cause for the decrease in the crime rate, anything that could possibly explain that decrease instead would be a possible alternative cause. An alternative cause would weaken the argument, which is all you need to do here - you don't need to prove the commissioner is wrong but merely give some contrary evidence that casts some new doubt on the commissioner's conclusion.

If more police were hired, as you said, that might lead to more arrests. Because this policy was enacted 2 years ago and it would have an increasing impact for 3 years (plus the time it would take for its effects to be felt), it is possible that more arrests in the initial stages of this program would deter violent criminals last year, so that the rate of violent crime would decrease. More arrests could deter in at least two ways - an arrest that led to prosecution and incarceration would take a "violent criminal" out of the general population, while any arrest no matter the results for the person arrested may cause potential violent criminals to reconsider. Because this is a policy more than 1 year old, the results of which could be delayed long enough for the violent crime rate to decrease last year by 15%, this policy provides a possible alternative cause for the decline in the rate of violent crime.

Robert Carroll
 tizwvu34
  • Posts: 20
  • Joined: Aug 15, 2016
|
#27797
I am wondering why answer choice D is not viable. If at the same time the law was enacted the definition was changed to include two previously unclassified violent crimes, couldn't the broadening of the definition account for the decreased percentage in violent crime, especially if those crimes rarely occur? Essentially, since there is a broader scope of what violent crime is, the percentage decrease could be to do the changing of the definition?

Or what if those two newly added crimes were decreasing on their own prior to them being classified as violent crimes?

Thank you for the clarification.
User avatar
 Jonathan Evans
PowerScore Staff
  • PowerScore Staff
  • Posts: 726
  • Joined: Jun 09, 2016
|
#27831
Hi, Tizwvu,

As the explanation notes above, if some crimes previously not violent were recategorized as violent crimes, a decrease in the violent crime rate would be yet more surprising. One needs to note that "rate" refers to how frequently such crimes occur/the total number of these crimes, not the percentage of crimes that are violent. Further, reclassifying crimes as violent would actually only increase the percentage of total crimes that are violent. With respect to your last point, it is certainly possible that if one were to suggest that these non-violent crimes now reclassified as violent were also retroactively classified as violent but were independently experiencing sharp decreases in their frequency that this situation could explain a decrease in the overall violent crime rate. However, you must consider the mental jujitsu we just did to arrive at that conclusion. The question asks you to find "which of the following, if true, most seriously weakens the police commissioner's argument?" Any time you have to make several additional assumptions to arrive at a given answer choice, you can be assured that this answer choice is not the credited response.
 adlindsey
  • Posts: 90
  • Joined: Oct 02, 2016
|
#31584
I actually looked at A as an alternate cause. I thought we could bring in new information with this question type?
 adlindsey
  • Posts: 90
  • Joined: Oct 02, 2016
|
#31585
I think I may have just made an unwarranted assumption. That of answer choice A, saying economic conditions was the cause.
 bk1111
  • Posts: 103
  • Joined: Apr 22, 2017
|
#34636
Hi, I am having trouble understanding why B is wrong. My thought process was that if judges were imposing harsh penalties prior to this new mandatory rule, it can serve as an alternate cause for the crime rate to drop by 15%.

After reviewing the question, I am trying to come up with explanations as to why it is does not weaken: (1) some crimes - it is vague while the stimulus refers to violent crimes, and (2) it had already done so for many years, but the crime rate drop by 15% was recent and therefore could not have been because of the harsh penalties imposed by the judges.

Is this correct? I am having trouble with Weaken questions in general so I am really trying to understand why each question is right or wrong. Thanks
 Steven Palmer
PowerScore Staff
  • PowerScore Staff
  • Posts: 35
  • Joined: Feb 21, 2017
|
#34654
Hi BK,

I think you're on the right track as to why (B) is incorrect. The conclusion is that the new sentencing law caused the drop in the rate of violent crime. (B) is too vague to be in the scope of the conclusion for the exact reasons you mentioned: namely that for many years the judges had been imposing the harsh penalties. Since it's been going on for a while, it would not explain why, out of the blue, the violent crime rate dropped 15%.

Answer choice (E) gives us a new cause for the effect, because the new officers were hired that year.

Hope this helps!
Steven
 mankariousc
  • Posts: 32
  • Joined: Feb 13, 2017
|
#34842
Hello!

While going through this question I was looking for an alternate cause. I thought A met that. Could you further explain why A doesn't work?

Thanks!

Get the most out of your LSAT Prep Plus subscription.

Analyze and track your performance with our Testing and Analytics Package.